Given a mean of 21 and a standard deviation of 1.2, what is the z-score of 21?

Answers

Answer 1
zero the z score of the mean is always zero

Related Questions

I’ll mark brainliest

Answers

Answer:

D

Step-by-step explanation:

Hi there!

We're given the equation y=-75x-50, which represents a submarine DESCENDING towards the ocean floor, where y is the depth in feet, and x is the number of minutes the submarine is descending

Since the submarine is DESCENDING, we can immediately eliminate A and C, which talk about the submarine ASCENDING

That leaves B and D

Looking at the given equation, y=-75x-50, -75 is the slope, or rate of change, and -50 is the y intercept, or the "beginning" (where the equation will "start")

Therefore, the submarine will start at -50 feet, or 50 feet below sea level

As x is the number of minutes the submarine is descending, that means that if the submarine travels 1 minute, it will descend 75 feet (-75*1=-75), at 2 minutes, it'll descend 150 feet (-75*2=-150), and so on

So that means the submarine must be descending at a rate of 75 feet per minute

Therefore D is the correct answer

Hope this helps! Good luck on your assignment :)

ABCD ~ QRST
· Find the missing side length, m

Answers

Since ABCD ~ QRST

AB/QR = AD/QT

=>6/m= 9/6

=> m = (6×6)/9 = 36/9 = 4

Answer:

m=4

Step-by-step explanation

since they are similar triangle .use these ratios

given:AB=6  , AD=9 , QR=m , QT=6

AB/QR=AD/QT

6/m=9/6

do cross multiplication

m*9=6*6

9m=36

m=36/9

m=4

therefore the value of m is 4

We want to construct a box with a square base and we currently only have 10m2 of material to use in construction of the box. Assuming that all material is used in the construction process, determine the maximum volume that the box can have.

Answers

Answer:

The maximum volume of the box is:

[tex]V =\frac{5}{3}\sqrt{\frac{5}{3}}[/tex]

Step-by-step explanation:

Given

[tex]Surface\ Area = 10m^2[/tex]

Required

The maximum volume of the box

Let

[tex]a \to base\ dimension[/tex]

[tex]b \to height[/tex]

The surface area of the box is:

[tex]Surface\ Area = 2(a*a + a*b + a*b)[/tex]

[tex]Surface\ Area = 2(a^2 + ab + ab)[/tex]

[tex]Surface\ Area = 2(a^2 + 2ab)[/tex]

So, we have:

[tex]2(a^2 + 2ab) = 10[/tex]

Divide both sides by 2

[tex]a^2 + 2ab = 5[/tex]

Make b the subject

[tex]2ab = 5 -a^2[/tex]

[tex]b = \frac{5 -a^2}{2a}[/tex]

The volume of the box is:

[tex]V = a*a*b[/tex]

[tex]V = a^2b[/tex]

Substitute: [tex]b = \frac{5 -a^2}{2a}[/tex]

[tex]V = a^2*\frac{5 - a^2}{2a}[/tex]

[tex]V = a*\frac{5 - a^2}{2}[/tex]

[tex]V = \frac{5a - a^3}{2}[/tex]

Spit

[tex]V = \frac{5a}{2} - \frac{a^3}{2}[/tex]

Differentiate V with respect to a

[tex]V' = \frac{5}{2} -3 * \frac{a^2}{2}[/tex]

[tex]V' = \frac{5}{2} -\frac{3a^2}{2}[/tex]

Set [tex]V' =0[/tex] to calculate a

[tex]0 = \frac{5}{2} -\frac{3a^2}{2}[/tex]

Collect like terms

[tex]\frac{3a^2}{2} = \frac{5}{2}[/tex]

Multiply both sides by 2

[tex]3a^2= 5[/tex]

Solve for a

[tex]a^2= \frac{5}{3}[/tex]

[tex]a= \sqrt{\frac{5}{3}}[/tex]

Recall that:

[tex]b = \frac{5 -a^2}{2a}[/tex]

[tex]b = \frac{5 -(\sqrt{\frac{5}{3}})^2}{2*\sqrt{\frac{5}{3}}}[/tex]

[tex]b = \frac{5 -\frac{5}{3}}{2*\sqrt{\frac{5}{3}}}[/tex]

[tex]b = \frac{\frac{15 - 5}{3}}{2*\sqrt{\frac{5}{3}}}[/tex]

[tex]b = \frac{\frac{10}{3}}{2*\sqrt{\frac{5}{3}}}[/tex]

[tex]b = \frac{\frac{5}{3}}{\sqrt{\frac{5}{3}}}[/tex]

Apply law of indices

[tex]b = (\frac{5}{3})^{1 - \frac{1}{2}}[/tex]

[tex]b = (\frac{5}{3})^{\frac{1}{2}}[/tex]

[tex]b = \sqrt{\frac{5}{3}}[/tex]

So:

[tex]V = a^2b[/tex]

[tex]V =\sqrt{(\frac{5}{3})^2} * \sqrt{\frac{5}{3}}[/tex]

[tex]V =\frac{5}{3} * \sqrt{\frac{5}{3}}[/tex]

[tex]V =\frac{5}{3}\sqrt{\frac{5}{3}}[/tex]

The maximum volume of the box which has a 10 m² surface area is given below.

[tex]\rm V_{max} = \dfrac{5}{3} *\sqrt{\dfrac{5}{2}}[/tex]

What is differentiation?

The rate of change of a function with respect to the variable is called differentiation. It can be increasing or decreasing.

We want to construct a box with a square base and we currently only have 10 m² of material to use in the construction of the box.

The surface area = 10 m²

Let a be the base length and b be the height of the box.

Surface area = 2(a² + 2ab)

  2(a² + 2ab) = 10

      a² + 2ab = 5

Then the value of b will be

[tex]\rm b = \dfrac{5-a^2}{2a}[/tex]

The volume of the box is given as

V = a²b

Then we have

[tex]\rm V = \dfrac{5-a^2 }{2a}* a^2\\\\V = \dfrac{5a - a^3}{2}\\\\V = \dfrac{5a}{2} - \dfrac{a^3}{2}[/tex]

Differentiate the equation with respect to a, and put it equal to zero for the volume to be maximum.

[tex]\begin{aligned} \dfrac{dV}{da} &= \dfrac{d}{da} ( \dfrac{5a}{2} - \dfrac{a^3}{2} ) \\\\\dfrac{dV}{da} &= 0 \\\\\dfrac{5}{2} - \dfrac{3a^2 }{2} &= 0\\\\a &= \sqrt{\dfrac{5}{2}} \end{aligned}[/tex]

Then the value of b will be

[tex]b = \dfrac{5-\sqrt{\dfrac{5}{2}} }{2*\sqrt{\dfrac{5}{2}} }\\\\\\b = \sqrt{\dfrac{5}{2}}[/tex]

Then the volume will be

[tex]\rm V = (\sqrt{\dfrac{5}{2}} )^2*\sqrt{\dfrac{5}{2}} \\\\V = \dfrac{5}{3} *\sqrt{\dfrac{5}{2}}[/tex]

More about the differentiation link is given below.

https://brainly.com/question/24062595

A roundabout is a one-way circular intersection.
About how many feet would a car travel if it drove
once around the roundabout? Round to the
nearest foot.

Answers

Answer:

[tex]471\:\mathrm{ft}[/tex]

Step-by-step explanation:

In one full rotation around the roundabout, the car is travelling a distance equal to the circumference, or the perimeter, of the circle. The circumference of a circle with radius [tex]r[/tex] is given by [tex]C=2r\pi[/tex]. In the diagram, the diameter is labelled 150 feet. By definition, the radius of a circle is exactly half of the diameter of the circle. Therefore, the radius must be [tex]\frac{150}{2}=75[/tex] feet. Thus, the car would travel [tex]2\cdot 75\cdot \pi=471.238898038=\boxed{471\:\mathrm{ft}}[/tex]

Identify an equation in slope-intercept form for the line parallel to y = 5x + 2 that passes through (-6, -1). O A. y = 5x – 29 O B. y = 1x+ 1 5 O C. y = 5x + 29 OD. y = -5x - 11​

Answers

Answer:

[tex]{\text{C. }y=5x+29[/tex]

Step-by-step explanation:

The slope-intercept form of a line is given by [tex]y=mx+b[/tex], where [tex]m[/tex] is the slope of the line and [tex]b[/tex] is the y-intercept.

The slopes of parallel lines are always equal. Therefore, the slope of the equation we want to find will also be [tex]5[/tex]. The equation of the line must be true for any point it passes through. Thus, we can plug in the point [tex](-6,-1)[/tex] to solve:

[tex]y=mx+b,\\-1=5(-6)+b,\\-1=-30+b,\\b=29[/tex]

Thus, the equation of the line is [tex]\boxed{\text{C. }y=5x+29}[/tex]

Find the area of the polygon shown. Enter the number into the box.
m 2
2 mi
10 mi
4 mi
12 mi
1
2

Answers

Answer:

See Explanation

Step-by-step explanation:

The question is incomplete, as the image of the polygon is not given.

I will answer this question with the attached image (similar to your question)

The attached polygon is a trapezoid of the following dimensions.

[tex]Height = 4ft[/tex]

Parallel sides

[tex]Side\ 1 = 4ft[/tex]

[tex]Side\ 2 = 4ft+1ft = 5ft[/tex]

So, the area is:

[tex]Area = \frac{1}{2} * (Side\ 1 + Side\ 2) * Height[/tex]

[tex]Area = \frac{1}{2} * (4ft + 5ft) * 4ft[/tex]

[tex]Area = \frac{1}{2} * 9ft * 4ft[/tex]

[tex]Area = 18ft^2[/tex]

Can someone help pleaseee

Answers

Answer:

Ŷ = 76.4064+5.4254X

0.786

Strong positive relationship

Score = 98

Step-by-step explanation:

Using technology, the linear model obtained by fitting the data is :

Ŷ = 76.4064+5.4254X

Where, slope = 5.4254

y = test score ; x = study time

The Correlation Coefficient obtained is 0.786 ; which depicts that there exist a strong positive relationship between the two variables.

Using the model; test score, if x = 4

Ŷ = 76.4064+5.4254(4)

Y = 98.108

Test score = 98

I need help with #5. Please help. I have to show all work so please explain

Answers

Answer:

d = 13

Step-by-step explanation:

Use distance formula:

[tex]d=\sqrt{(x_2-x_1)^2+(y_2-y_1)^2}\\d=\sqrt{(5-0)^2+(5-(-7))^2}\\d=\sqrt{5^2+12^2}\\d=\sqrt{25+144}\\d=\sqrt{169}\\d=13[/tex]

a + b = 300 pls help i cant find out the answer

Answers

Answer:

a= 250

b= 50

250 + 50 = 300

Step-by-step explanation:

There's many solutions but this was the first one I could come up with.

Answer:

my opinion is seince a+b=300 then the sqaure of 300= 17.3?

Step-by-step explanation:

This table on a package of dog food tells how much to feed a dog, depending on its weight.  Weight of Dog (pounds)153045 Amount of Food (scoops)246 The amount of food in scoops (s) is related to the weight of the dog in pounds (p) by the equation s = kp. What is k?​

Answers

9514 1404 393

Answer:

  k = 2/15

Step-by-step explanation:

We can solve the given equation for k:

  k = s/p . . . . . . divide the given equation by p on both sides

Using the first values from the table (15 pounds, 2 scoops), we have ...

  k = 2/15

The value of k is 2/15.

Will mark Brainlest help plsssss​

Answers

Answer:

45 is answer I guess cuz my teacher taught me just like that

What is the volume of a rectangular prism
8 inches long, 3 inches wide, and 5 inches high?
A
120 cubic inches
B
220 cubic inches
16 cubic inches
158 cubic inches

Answers

Answer:

A; 120 cubic inches

Step-by-step explanation:

Let us start with the formula of the volume of a rectangular prism,[tex]V=l*w*h[/tex], where l represents the length of the prism, w represents the width of the prism, and h represents the height of the prism. It is given to us that h =5 inches, w =3 inches, and l =8 inches. Let's plug the values in:

[tex]V= 8*3*5\\V=120[/tex]

A. The volume of the rectangular prism is 120 cubic inches.

I hope this helps! Let me know if you have any questions :)

Abigail is using blocks to build a tower. The blocks are 3 inches, 4 inches, and 8 inches tall. She has stack 3 blocks. How many different heights are possible for the tower?

Answers

9514 1404 393

Answer:

  10

Step-by-step explanation:

Possible tower heights using 3 blocks are ...

  {9, 10, 11, 12, 14, 15, 16, 19, 20, 24}

There are 10 different heights possible.

_____

Each block can be used 1, 2, or 3 times.

Using a 3 in block as the smallest, we have ...

  3+3+3 = 9

  3+3+4 = 10

  3+3+8 = 14

  3+4+4 = 11

  3+4+8 = 15

  3+8+8 = 19

Using a 4-in block as the smallest, we have ...

  4+4+4 =12

  4+4+8 = 16

  4+8+8 = 20

And ...

  8+8+8 = 24

What is the value of x?
Enter your answer as a decimal in the box. Round only your final answer to the nearest tenth.
x =___m

Answers

Answer:

<W=180 - (30+81)

<W=69°

Using Sine rule to evaluate x

x/sin30 = 19/sin69

x= 19sin30/sin69

x= 10.2m ( Nearest tenth)

mary drinks 24 ounces of juice a day . lena drinks three times as much. how many ounces do they drink together?​

Answers

Answer:

96 oz.

Step-by-step explanation:

Mary drinks 24 ounces a day Lena drinks 3 times a much

24 x 3 = 72

72 + 24 = 96

Answer:

They dinks ounces of juice together = 96 ounces.

Step-by-step explanation:

Given that :-

Mary drinks 24 ounces of juice a day.Lena drinks three times as much.

To find :-

How many ounces do they drink together ?

Solution :-

Mary drinks 24 ounces of juice a day = 24 ounces.

Lena drinks three times much than mary = 3 × 24 ounces = 72 ounces

They drinks ounces together = mary drinks ounces of juice + lena drinks ounces of juice

= 24 ounces + 72 ounces

Hence , They dinks ounces of juice together = 96 ounces.

The nth term of a sequence is 5n.
Work out the 10th term of this sequence.

Answers

Answer:

The 10th term is 50

Step-by-step explanation:

5(10) = 50

Please can somebody help I am not very good at maths

Answers

Answer:

z = 3a + 4

Step-by-step explanation:

Add 4 to both sides

Write an equation of the line with a slope of 2/3
and -8

Answers

y = 2/3 x - 8
I assume the -8 was the y intercept?

below is a table showing the investment and the investment period of ​

Answers

Answer:

hey. pls complete your question.

Will mark Brainlest please answer. find the value of a,b.
,p,q from the equal order pairs​

Answers

Step-by-step explanation:

Question-1:

by order pair we obtain:

[tex] \displaystyle \begin{cases} \displaystyle 3p = 2p - 1 \dots \dots i\\2q - p = 1 \dots \dots ii\end{cases}[/tex]

cancel 2p from the i equation to get a certain value of p:

[tex] \displaystyle \begin{cases} \displaystyle p = - 1 \\2q - p = 1 \end{cases}[/tex]

now substitute the value of p to the second equation:

[tex] \displaystyle \begin{cases} \displaystyle p = - 1 \\2q - ( - 1) = 1 \end{cases}[/tex]

simplify parentheses:

[tex] \displaystyle \begin{cases} \displaystyle p = - 1 \\2q + 1= 1 \end{cases}[/tex]

cancel 1 from both sides:

[tex] \displaystyle \begin{cases} \displaystyle p = - 1 \\2q = 0\end{cases}[/tex]

divide both sides by 2:

[tex] \displaystyle \begin{cases} \displaystyle p = - 1 \\q = 0\end{cases}[/tex]

question-2:

by order pair we obtain:

[tex] \displaystyle \begin{cases} \displaystyle 2x - y= 3 \dots \dots i\\3y= x + y \dots \dots ii\end{cases}[/tex]

cancel out y from the second equation:

[tex] \displaystyle \begin{cases} \displaystyle 2x - y= 3 \dots \dots i\\ x = 2y \dots \dots ii\end{cases}[/tex]

substitute the value of x to the first equation:

[tex] \displaystyle \begin{cases} \displaystyle 2.2y-y= 3 \\ x = 2y \end{cases}[/tex]

simplify:

[tex] \displaystyle \begin{cases} \displaystyle 3y= 3 \\ x = 2y \end{cases}[/tex]

divide both sides by 3:

[tex] \displaystyle \begin{cases} \displaystyle y= 1 \\ x = 2y \end{cases}[/tex]

substitute the value of y to the second equation which yields:

[tex] \displaystyle \begin{cases} \displaystyle y= 1 \\ x = 2 \end{cases}[/tex]

Question-3:

by order pair we obtain;

[tex] \displaystyle \begin{cases} \displaystyle 2p + q = 2 \dots \dots i\\3q + 2p = 3 \dots \dots ii\end{cases}[/tex]

rearrange:

[tex] \displaystyle \begin{cases} \displaystyle 2p + q = 2 \\2p + 3q= 3 \end{cases}[/tex]

subtract and simplify

[tex] \displaystyle \begin{array}{ccc} \displaystyle 2p + q = 2 \\2p + 3q= 3 \\ \hline - 2q = - 1 \\ q = \dfrac{1}{2} \end{array}[/tex]

substitute the value of q to the first equation:

[tex] \displaystyle 2.p+ \frac{1}{2} = 2[/tex]

make q the subject of the equation:

[tex] \displaystyle p = \frac{3}{4} [/tex]

hence,

[tex] \displaystyle q = \frac{1}{2} \\ p = \frac{3}{4} [/tex]

Answer:

see above

............

Which point is the center of the circle that contains the vertices of a triangle?

Answers

The circumcenter is the center of the circle that contains the vertices of a triangle

How to determine the point?

When a triangle is inscribed in a circle, the vertices of the triangle touch the circumference of the circle

A line drawn through the center of the circle and passes through each of the triangle vertex is its circumcenter.

Hence, the name of the required point is the circumcenter

Read more about circumcenter at:

https://brainly.com/question/14368399

#SPJ2

Answer:

B. The point of intersection of the perpendicular bisectors of the side

Step-by-step explanation:

definition of circumcenter as the previos question answered

what's the easiest way to answer how I know the answer pls?​

Answers

Answer: Table C

Explanation: The X values match up with those on the graph!

The function of f(x) = 3x + 2 has a domain of -3 < x < 5. What is the domain of f-1(x)?

Answers

Answer:    -7 < x < 17

====================================================

Explanation:

Plug in the lower bound of the domain, which is x = -3

f(x) = 3x+2

f(-3) = 3(-3)+2

f(-3) = -9+2

f(-3) = -7

If x = -3, then the output is y = -7. Since f(x) is an increasing function (due to the positive slope), we know that y = -7 is the lower bound of the range.

If you plugged in x = 5, you should find that f(5) = 17 making this the upper bound of the range.

The range of f(x) is -7 < y < 17

Recall that the domain and range swap places when going from the original function f(x) to the inverse [tex]f^{-1}(x)[/tex]

This swap happens because how x and y change places when determining the inverse itself. In other words, you go from y = 3x+2 to x = 3y+2. Solving for y gets us y = (x-2)/3 which is the inverse.

-----------------------

In short, we found the range of f(x) is -7 < y < 17.

That means the domain of the inverse is -7 < x < 17 since the domain and range swap roles when going from original to inverse.

The domain of the resulting function exists on all real values that is the domain is -∞ < f-1(x) < ∞

How to find the domain of an inverse function?

The domain of a function are the independent values of the function for Which it exists.

Given the function f(x) = 3x + 2

Find its inverse

y = 3x + 2

Replace x with y

x = 3y + 2

Make y the subject of the formula:

3y = x - 2
y = (x-2)/3

The domain of the resulting function exists on all real values that is the domain is -∞ < f-1(x) < ∞
Learn more on domain here: https://brainly.com/question/26098895

y=6x/5 +27 find y-intercept and slope

Answers

Answer:

General equation of a line is given by y = mx +c, where m is the gradient /slope, c is the intercept. To find for the intercept on y- axis, put x = 0.[tex]y = \frac{6(0)}{5 } + 27 \\ y = \frac{0}{5} + 27 \\ y = 27 \\ therefore \: the \: intercept \: on \: y \: is \: 27 \\ [/tex]By comparison, [tex]y = mx + \: c \\ y = \frac{6}{5} x \: +27 \\ m = \frac{6}{5 \: } \: \\ hence \: slope \: is \: \frac{6}{5} [/tex]

The rectangular ground floor of a building has a perimeter of 780 ft. The length is 200 ft more than the width. Find the length and the width.
The length is ___ and the width is ___

Answers

Answer:

perimeter of the rectangular ground floor

=2(length+width)

length=X+200

width=X

=2(X+200+X)

=4x+400

4x+400 =780

4x =780-400

4x =380

x =95

width=95 feet

length=95+200

=295 feet

Find the equation of a sphere if one of its diameters has endpoints: (-14. -3, -6) and (-4, 7, 4) Note that you must move everything to the left hand side of the equation and that we desire the coefficients of the quadratic terms to be 1.

Answers

Answer:

[tex]x^2+y^2+z^2-18x-4y+2z-21=0[/tex]

Step-by-step explanation:

From the question we are told that:

Diameters has endpoints: [tex](-14. -3, -6) & (-4, 7, 4)[/tex]

Generally the equation for Center of The sphere is mathematically given by

 [tex]C=(\frac{-14+(-4)}{2},\frac{-3+(7)}{2},\frac{-6+(4)}{2})[/tex]

 [tex]C=(9,2,-1)[/tex]

Generally the equation for Radius of the sphere is mathematically given by

 [tex]R=\sqrt{(9-2)^2+(2-9)^2+(-1-2)^2}[/tex]

 [tex]R=\sqrt{107}[/tex]

Therefore the Equation of the Sphere is

 [tex](x-9)^2+(y-2)^2+(z+1)^2=107[/tex]

 [tex](x^2-18x+81)+(y^2-4y+4+(z^2+2z+1))=107[/tex]

 [tex]x^2+y^2+z^2-18x-4y+2z-21=0[/tex]

Simply the following ratio 1000:540:780

Answers

This is my workings

Write the equation in standard form for the circle with center (-3,3) and radius 3

Answers

Answer:

(x+3)^2+(y−3)^2=9

Step-by-step explanation:

The equation for a circle is given by

(x−h)^2+(y−k)^2=r^2  where (h,k) is the center and r is the radius

(x− -3)^2+(y−3)^2=3^2

(x+3)^2+(y−3)^2=9

Determine if the two triangles are congruent. If they are, State how you know. NO LINKS!!!!! Show work please. Part 3c​

Answers

Answer:

7.  Not enough information to determine congruency

8.congreunt by AAS

Step-by-step explanation:

7.  We know one side and the vertical angles between the triangles.  That is not enough to determine the triangles are congruent

8.  We know one angle and one side are congruent.  The angle between are vertical angles and they are congruent.    We know two angles and one side, so they are congruent by AAS

Which trig ratio can be used to find the measure of angle A?

Answers

Answer:

arc cosine (4/5)

(the third answer)

Step-by-step explanation:

Other Questions
help please! the question is in the picture below for reference when answering! thank you so much! Match each expression it to an equivalent expression Math help please urgent This information relates to Blossom Co..1. On April 5, purchased merchandise from Sunland Company for $28,800, terms 4/10, n/30.2. On April 6, paid freight costs of $620 on merchandise purchased from Sunland Company.3. On April 7, purchased equipment on account for $34,200.4. On April 8, returned $3,500 of April 5 merchandise to Sunland Company.5. On April 15, paid the amount due to Sunland Company in full.1. Prepare the journal entries to record the transactions listed above on Blossom Co.s books. Blossom Co. uses a perpetual inventory system.2. Assume that Blossom Co. paid the balance due to Sunland Company on May 4 instead of April 15. Prepare the journal entry to record this payment. On Tuesday, a local hamburger shop sold a combined total of 576 hamburgers and cheeseburgers. The number of cheeseburgers sold was three times the number of hamburgers sold. How many hamburgers were sold on Tuesday? Whats the answer of this one The greatest danger facing by the most endangered species isCapture for tradesDiseaseSpecies loss I need help please ASAP Which expression is equivalent to the following fraction? Calculate the volume of a cone with: 11cm height and 6cm radius determine the value of a x1-3x3=-3 and 2x1+ax2-x3=-2 has no solution Hey, can someone help me with this fast thanks!! Find the 40th term of the arithmetic sequence with a = 150 and a common difference of d =-4.1.-62.63.-24.2 she cooked rice passive 8. Why are the proteins produced by a GMO crop monitored and evaluated? O A. To determine whether the organism will cause an allergic reaction O B. To evaluate whether the Bt toxin is produced O C. To measure hormones present in dairy products O D. To examine the need for additional fertilizers Jamal was restless early in the morning. He has a million thoughts going through his mind. Which part of bipolar disorder is he experiencing, and what should he take?a. He is experiencing mania, and he should be taking antidepressants.b. He is experiencing mania, and he should be taking antianxiety.c. He is experiencing depression, and he should be taking antidepressants.d. He is experiencing depression, and he should be taking antianxiety drugs. company paid $48 for 2 cases of printer paper. Each case contained 12 packages of paper. Next month the companys office manager needs to order 180 packages of the same paper. If the price per package does not change, what would be the total c Mr sharma can do 1/15part of piece of work in 1 day he worked for 3 days and left if Vishnu finished the remaining work how much work is done by bishnu? Please help Ill give brainliest Grant is working at a veterinarian's office, and he needs to give a dose of medicine to a25 kg dog. If a 10 kg dog needs 18 mL of the same medicine, how many milliliters ofmedicine does the 25 kg dog need?